How is this a strengthen with Sufficient Premis...
This looks more like an errors of reasoning question the way the question is being addressed.
Tyler808 on April 28, 2022
  • October 2015 LSAT
  • SEC4
  • Q5
2
Replies
Difference between A and C
I do not understand the differences between A and C, and why A is correct and C is not. Could thi...
alannadels on April 21, 2022
  • October 2015 LSAT
  • SEC4
  • Q22
1
Reply
Why is D incorrect?
Wouldn't participating in other programs besides chess contribute as an alternative explanation t...
connordelacruz on March 26, 2022
  • October 2015 LSAT
  • SEC4
  • Q21
1
Reply
Question help
I identified this question as a point at issue question. I went on to be tricked by the question...
Walker on March 22, 2022
  • October 2015 LSAT
  • SEC4
  • Q10
4
Replies
Why D?
It seems like the editor directly counters D by saying it will result in the same volume being le...
Christopher on February 6, 2022
  • October 2015 LSAT
  • SEC4
  • Q23
5
Replies
Why not A?
Wouldn't a negation of A also make this argument fall apart? If the hairless dogs were found else...
dannyod on January 26, 2022
  • October 2015 LSAT
  • SEC4
  • Q18
7
Replies
Question
Why is the answer A instead of D? Thanks!
claire_crites on January 26, 2022
  • October 2015 LSAT
  • SEC4
  • Q22
5
Replies
Please explain why the answer is C
I thought for Paradox questions, the answer needs to establish why the change happened. Answer C ...
Andrew on May 12, 2021
  • October 2015 LSAT
  • SEC4
  • Q3
1
Reply
Answer B
Isn't answer choice B the answer if the question stem is asking what is the assumption that is re...
yassmenaraim on January 8, 2021
  • October 2015 LSAT
  • SEC4
  • Q14
1
Reply
Why C over B?
Why not B?
samxinghaoli on September 6, 2020
  • October 2015 LSAT
  • SEC4
  • Q21
3
Replies
Help
Can someone explain this question? I thought it was A because it does appear to use the word aver...
SamBaucom on June 20, 2020
  • October 2015 LSAT
  • SEC4
  • Q13
4
Replies
Question
Why is this A and not D? Thanks!
claire_crites on June 7, 2020
  • October 2015 LSAT
  • SEC4
  • Q17
2
Replies
Why not E?
Hi! I'm a little unclear as to why the answer couldn't be E. What if there were microdiamonds tha...
DavidW on March 17, 2020
  • October 2015 LSAT
  • SEC4
  • Q19
3
Replies
Why is C correct? Why is E incorrect?
Thanks
Ryan-Mahabir on December 8, 2019
  • October 2015 LSAT
  • SEC4
  • Q9
3
Replies
B
Why doesn't B work? the passage first talks about real average income as the income adjusted for ...
tomgbean on December 4, 2019
  • October 2015 LSAT
  • SEC4
  • Q13
1
Reply
Why E and not B?
I was torn between them and ended up selecting B. Thanks!
cjahangiri on December 3, 2019
  • October 2015 LSAT
  • SEC4
  • Q18
1
Reply
B vs. C
Can someone please explain?
masonnavarrete on November 23, 2019
  • October 2015 LSAT
  • SEC4
  • Q16
1
Reply
Why is B correct? Why is C incorrect?
Thanks
Ryan-Mahabir on November 8, 2019
  • October 2015 LSAT
  • SEC4
  • Q14
1
Reply
Why is D correct? Why is C incorrect?
Thanks
Ryan-Mahabir on October 27, 2019
  • October 2015 LSAT
  • SEC4
  • Q24
1
Reply
Please need help
Explain the answer
ali on October 15, 2019
  • October 2015 LSAT
  • SEC4
  • Q25
1
Reply